Patho Final Exam

¡Supera tus tareas y exámenes ahora con Quizwiz!

The nurse is caring for a client who receives hemodialysis. The nurse knows that hemodialysis involves movement of charged or uncharged particles along a concentration gradient. Which function best describes this process?

diffusion

After exposure to poison ivy, the client has hives on his trunk and extremities. In an effort to decrease his immune response the practitioner places him on which medication?

glucocorticoids

Nutrition students are studying the nervous system, which has a high rate of metabolism. What is its major fuel source?

glucose

A 30-year-old male arrives with these vital signs: blood pressure 50/30, pulse 100, respiratory rate 12, temperature 101. The nurse interprets these results as the client is in shock based on which reading?

low blood pressure

The emergency room doctor suspects a client may have bacterial meningitis. The most important diagnostic test to perform would be:

lumbar puncture

The nurse is administering a unit of packed red blood cells to a client and piggybacks the unit of blood through a solution of 0.9% NaCl. Blood cells placed in a solution of 0.9% saline will do which of the following?

neither shrink nor swell

The nurse is teaching a class of pregnant woman about nutrition, emphasizing their need for folic acid. The nurse's rationale for this relates to prevention of which birth defect?

neural tube defects

The nurse is learning about mitochondrial DNA mutations. She learns that they generally affect which tissues and organs?

neuromuscular

Following an injury resulting in a small cut from a knife, the first cells to go to the area of the cut would be the:

neutrophils

A school nurse is teaching a class on immunity. Which statement contains an accurate explanation about cellular defenses?

neutrophils engulf invading organisms where lysosomes break them down.

The nurse is caring for the following group of client. Select the client most likely to be diagnosed with respiratory alkalosis.

A 26-year-old female with anxiety who has been hyperventilating.

A client who is recovering from burn injuries is discussing his prognosis with a physician. Which teaching point about expectations for healing should the physician offer?

"you may find that the scar is a bit smaller than the area of the wound."

A 51-year-old female client who is 2 days postoperative in a surgical unit of a hospital is at risk of developing atelectasis as a result of being largely immobile. Which teaching point byre nurse is most appropriate?

"you should breathe deeply and cough to help your lungs expand as much as possible which you're in bed."

A client who has been receiving cortisol as treatment for an illness states he "is constantly developing colds and fevers." What is the nurse's best response to the client?

"Cortisol inhibits the function of the immune system, making you more susceptible to disease."

Which student statement demonstrates a sound understanding of the cellular processes of hypertrophy and hyperplasia?

"I know that cells like neurons have little capacity for hyper plastic growth."

A client diagnosed with type 2 diabetes has been instructed about managing his condition with diet. The nurse determines further teaching when the client states:

"I must avoid all candies and cookies, but can eat unlimited amounts of pasta and bread."

A nurse is teaching parents of a child with scarlet fever. Which statement indicates that the parents understand the treatment for the child?

"I will give the antibiotic for the full 10 days."

A nurse is teaching a parent group about the importance of regular childhood vaccines. The nurse evaluates that the teaching is successful based on which of the following statements?

"I will have my baby vaccinated as scheduled to reduce the senseless spread of vaccine-preventable illnesses."

A client is diagnosed with Addison's disease. What statement by the client indicates an understanding of the discharge instruction by the nurse?

"I will have to take my medication for the rest of my life."

Which teaching point would be most appropriate for a group of older adults who are concerned about their cardiac health?

"The plaque that builds up in your heart vessels obstructs the normal flow of blood and can even break loose and lodge itself in a vessel."

A client has been exercising his arms repeatedly. Which statement by the nurse explains the increase in the size of the arms?

"Your cells have hypertrophied in response to increased workload."

A 35-year-old client is diagnosed with acute kidney injury (AKI) and is started on hemodialysis. The client is concerned with the diagnosis and wants to know what to expect in the progression of this disorder. Which statement best addresses the client's concern?

"acute kidney injury is abrupt in onset and often reversible if recognized early and treated appropriately."

The father of a 2-year-old boy recently diagnosed with hemophilia A asks the nurse how to prevent complications for his son. The best response would be:

"avoid administering aspirin and non steroidal anti-inflammatory drugs."

The parent of an infant who developed hydrocephalus while in utero is very concerned that the child will have significant intellectual dysfunction. The best response to the parent would be:

"because the skull sutures are not fused there may be no brain damage."

A client who has recently diagnosed with cancer and is worried that his cancer might spread asks the nurse how cancer spreads? What would be the nurse's best response?

"cancer spreads through the lymph channels and the blood vessels."

A pregnant woman is concerned about acquiring toxoplasmosis. The most important information for the nurse to give the client would be:

"do not clean your cat's litter box."

The family of an older adult client is wondering why his "blood counts" are not rising after his last GI bleed. They state, "he has always bounced back after one of these episodes, but this time it isn't happening. Do you know why?" The nurse will respond based on which pathophysiologic principle?

"due to stress, the red blood cells of older adults are not replaced as promptly as younger people."

The nurse is caring for a female client with cholelithiasis. When teaching the client about the disease, the nurse includes which of the points?

"gallstones have developed, which are typically composed of cholesterol."

A client who has donated a kidney to his son asks what will happen to his remaining kidney. What is the nurse's best response?

"hypertrophy will occur as the kidney's workload increases"

The mother of a newborn infant questions why her baby needs a vitamin K injection immediately after birth. The best response by the nurse would be:

"infants are not born with the normal intestinal bacteria that synthesize vitamin K for clotting."

A client receives a phone call from her gynecologist's office nurse and is informed that her Pap test results identified mild dysplasia. The best explanation for the nurse to provide would be:

"minor degrees of dysplasia are associated with chronic irritation or inflammation. However, we will need to do additional diagnostic studies to confirm the diagnosis."

On a routine physical exam visit, the physician mentions that he hears a new murmur. The client gets worried and asks, "what does this mean?" They physician responds:

"one of your heart valves is not opening properly. We need to do an echocardiogram to see which valve is having problems."

As part of their orientation to a cardiac care unit, a group of recent nursing graduates is receiving a refresher in a cardiac physiology from the unit educator. Which teaching point best captures a component of cardiac function?

"the diastolic phase is characterized by relaxation of ventricles and their filing with blood."

Which response by the nurse would be most accurate when a client with cancer asks why he feels so fatigued?

"the disease and treatment can cause you to feel like this."

A child has been home from camp for 2 weeks and complains of sore throat, low-grade fever, and enlarged cervical lymph nodes. After testing, it is determined the child has Epstein-Barr virus (EBV)-associated infectious mononucleosis. The parent asks "How did my child acquire this type of infection?" Which is the best response by the nurse?

"the infection is acquired primarily through contact with infected oral secretions."

A client in the nursing home wonders why he is having these signs and symptoms of left-sided heart failure. Which explanation will the nurse give the client?

"the left ventricle is having problems pumping blood forward, and this is caring blood to back up into your lungs."

Which statement would a nurse tell a client that best describes a lab finding of metaplasia?

"this is a change in cell form as it adapts to increased work demands or threats to survival."

A client tells the health care provider he has heard several myths about viruses. The most appropriate information for the nurse to provide would be:

"viruses can produce symptoms of disease months to years later"

A 51-year-old male professional is in the habit of consuming 6 to 8 rum and cokes each evening after work. He assures the nurse practitioner who is performing his regular physical exam that his drinking is under control and does not have negative implications for his work or family life. How could the nurse best respond to the client's statement?

"when your body has to regularly break down that much alcohol, your blood and the functional cells in your liver accumulate a lot of potentially damaging toxic by products."

A client experiencing asymptomatic UTIs while pregnant. The client asks the nurse if this places her at any risk for complications. Which would be the best response by the nurse?

"you are at risk for developing acute pyelonephritis."

A health care provider for an obese male who has a history of diabetes and hypertension reports that the client's glomerular filtration rate (GFR) is 51 mL/min with elevated serum creatinine levels. Which statement by the health care provider will likely answer the client's question about the result?

"your chronic kidney disease has likely been caused by your diabetes and high blood pressure."

A client has been taking up to 2400 mg of ibuprofen per day for chronic pain following a motor vehicle accident. The client is diagnosed with chronic analgesic nephritis. The client states, "I thought that taking too many drugs hurt your liver if anything, not your kidneys." What is the most appropriate response to the client's statement?

"your kidneys are vulnerable to damage because of how much blood flows through them and the fact that they break down many drugs."

A client is admitted with cold exposure. Which assessment findings will the nurse explain as physiologic changes seen by the client? SELECT ALL THAT APPLY.

-"decrease in blood flow leads to hypoxia." -"hypoxia leads to tissue injury and death." -"vasoconstriction leads to decreased blood flow to tissues."

Which clients are showing manifestations of infection? SELECT ALL THAT APPLY.

-a 2-month-old, temperature 38.3 (100.4), lethargy, poor feeding, and cyanosis. -a 75-year-old, temperature 37.3 (99.2) declining mental status, weakness, and fatigue. -a 25-year-old, temperature 40 (104), sweating, shivering, states generalized pain.

The nurse is caring for a group of clients and deciding who should receive prophylaxis for venous thromboembolism (VTE). Which clients should the nurse identify at high risk for VTE? SELECT ALL THAT APPLY.

-a 65-year-old who had a stroke and is now immobile -a 25-year-old on oral contraceptive who smokes -a 45-year-old with a fractured patella

The nurse recognizes that which individuals have the greatest risk for the development of chronic kidney disease (CKD)? SELECT ALL THAT APPLY.

-a client with a recent diagnosis of type 2 diabetes who does not monitor blood sugar -a 42-year-old client with uncontrolled hypertension averaging 170/94 mm Hg.

The nurse is planning care for a group of clients. Which clients should the nurse prioritize as "at risk" for hypercalcemia and advocate for monitoring calcium levels? SELECT ALL THAT APPLY

-a client with prolonged immobility -a client with hypophosphatemia -a client who has breast cancer with bone metastasis

A nurse is teaching a group of clients about causes/risk factors for cancer, which include which factors? SELECT ALL THAT APPLY.

-age -environment -genetics -heredity

A male with a history of excess alcohol consumption for 25 years is admitted with cirrhotic liver disease. To assess the amount of damage to the liver, which lab tests of hepatobiliary functions should be monitored? SELECT ALL THAT APPLY.

-alanine aminotransferase (ALT) -Y-glutamyultransferase (GGT) -aspartate aminotransferase (AST)

The nurse is teaching a group of nursing students about hepatitis. Which of these does the nurse include in teaching about the causes of hepatitis? SELECT ALL THAT APPLY.

-alcohol consumption -viruses -autoimmune disorders

A widow who lost her husband a few weeks ago is having trouble with insomnia. When visiting with her health care provider, the provider suggests a prescription to help her regain a normal circadian pattern. This is based on the fact the interruption of sleep-wake cycles can cause which problems? SELECT ALL THAT APPLY.

-alterations in immune function that can result in an infection. -an increased risk in accidents when sleep deprived, similar to those under the influence of alcohol.

The nurse teaches a client that genetic information is responsible for which body function? SELECT ALL THAT APPLY.

-appearance -function of body cells -reaction of drugs -response to the environment

A client admitted to the hospital with elevated blood glucose is diagnosed with type 1 diabetes mellitus. What characteristics commonly differentiate type 1 diabetes mellitus from type 2 diabetes mellitus? SELECT ALL THAT APPLY.

-autoimmune beta cell damage -abrupt onset of symptoms -onset before age 20

A client is experiencing hepatorenal syndrome. Which manifestations will the nurse assess in this client? SELECT ALL THAT APPLY.

-azotemia -increased serum creatinine -oliguria -ascites

The nurse is caring for a client who has a tumor that results in excess levels of catecholamines being released. Which assessments will the nurse prioritize in this client's plan of care? SELECT ALL THAT APPLY.

-blood glucose -heart rate and rhythm -blood pressure

The nurse is caring for a client who has a tumor that results in excess levels of catecholamines being released. Which assessments will the nurse prioritize in the client's plan of care? SELECT ALL THAT APPLY.

-blood glucose -blood pressure -heart rate and rhythm

A 25-year-old client with cystic fibrosis presents to the clinic in obvious respiratory distress. Following physical exam, the health care provider suspects bronchiectasis based on which assessment findings? SELECT ALL THAT APPLY.

-blood-tinged sputum -copious amounts of foul-smelling purulent sputum -wheezing throughout the lung fields

Select the most appropriate statements that describes the "stress response." SELECT ALL THAT APPLY.

-brings the body to a stable state -a normal response

The body regulates the pH of its fluid by what mechanism? SELECT ALL THAT APPLY.

-chemical buffer systems of the body fluids -the lungs -the kidneys

Which disorders and/or treatments would be classified as potential causes of thrombocytopenia that affect bone marrow function? SELECT ALL THAT APPLY.

-chemotherapy -radiation therapy

A client with cystic fibrosis reports sinus congestion and coughing up thick greenish mucus. What interventions will the nurse implement? SELECT ALL THAT APPLY.

-chest physiotherapy -antibiotic therapy

A client is admitted with the diagnosis of obstructive jaundice. Which assessment findings would the nurse expect to see in this client? SELECT ALL THAT APPLY.

-clay-colored stools -dark urine -elevated conjugated bilirubin levels -severe itching

The nurse assessing a client with a traumatic brain injury assesses for changes in which neurologic component? SELECT ALL THAT APPLY.

-cognition -sensory function -motor function -level of consciousness

A nurse is planning a community education program on lifestyle modification to manage hypertension. Which topic should be included in the teaching plan? SELECT ALL THAT APPLY.

-consume a diet rich in fruits, vegetables, and low-fat dairy products -stop smoking -reduce dietary sodium intake -limit alcohol consumption

A nurse assessing an older adult for signs and symptoms of infection in the absence of a fever should assess for which of the following? SELECT ALL THAT APPLY.

-decreased mental status -change in functional capacity -fatigue

The nurse is admitting a 90-year-old client with a history of heart failure, coronary artery disease, prostate cancer, diabetes, and hypercholesterolemia. Which conditions in this client's history place the person at high risk for thromboembolic complications? SELECT ALL THAT APPLY.

-diabetes -coronary artery disease -hypercholesterolemia

There can be many reasons for a client to present with hypoxemia. For a client's PO2 to fall, a respiratory disease is usually involved. Often, clients have involvement from more than one mechanism. Which factors will result in hypoxemia? SELECT ALL THAT APPLY.

-dysfunction of neurologic system -disease in respiratory system -decreased oxygen in air

The nurse is assessing a female client with a hemoglobin of 6.8 g/dL (68 g/L). Which symptoms would the nurse expect to find? SELECT ALL THAT APPLY.

-dyspnea -faintness -headache

Which manifestations of leukemia are directly related to a decrease in thrombocytes. SELECT ALL THAT APPLY.

-ecchymosis -petechiae

A client had developed jaundice. The nurse recognizes that the jaundice may have resulted from which cause? SELECT ALL THAT APPLY.

-excessive destruction of red blood cells -impaired uptake of bilirubin by the liver cells -decreased conjugation of bilirubin -obstruction of bile flow

While teaching a preconception workshop for young adults, the nurse knows that the participants understand the educational teaching when they state which agents are teratogenic? SELECT ALL THAT APPLY.

-having a glass of wine with dinner -casual use of cocaine -the use of warfarin

The nurse is studying hemostasis. She knows that which statements are true?

-hemostasis results in the formation of a platelet plug. -hemostasis occurs in established stages. -hemostasis refers to the body's ability to control bleeding. -vessel constriction or spasm is a factor in effective effective hemostasis.

A neonate is undergoing phototherapy for the treatment of jaundice and accompanying high levels of bilirubin. Which causes are the likely factors responsible for neonatal hyperbilirubinemia? SELECT ALL THAT APPLY.

-high bilirubin production -limited ability to excrete bilirubin

A teenager is diagnosed with hypertension. The nurse knows that risk factors for hypertension in children and adolescents include which of these? SELECT ALL THAT APPLY.

-high salt consumption -obesity -inactive lifestyle

Which factors increase the risk that a client will develop chronic obstructive pulmonary disease (COPD)? SELECT ALL THAT APPLY.

-history of tobacco use -history of asthma

Which manifestations are most common in clients with COPD that is predominantly emphysema? SELECT ALL THAT APPLY.

-hyperresonance -increased A-P diameter

A client seen in the emergency department after a motor vehicle accident complains of increasing shortness of breath. Which manifestations support the diagnosis of a possible left tension pneumothorax? SELECT ALL THAT APPLY.

-hyperresonance on the left -diminished breath sounds on the left -subcutaneous emphysema -tracheal deviation to the right

The single parent is grieving the recent loss of his only child and has limited coping abilities. Which health problem may be induced by this acute stress? SELECT ALL THAT APPLY.

-hypertension -obesity -anxiety -migraines

The nurse is teaching a client who has been place on an angiotensin receptor blocker. The nurse explains this medication will block the renin-angiotensin-aldosterone system (RAAS) which, when activated, causes which assessment findings? SELECT ALL THAT APPLY.

-increase in peripheral vascular tone -renal retention of sodium and water

The nurse is teaching a client who has been placed on an angiotensin receptor blocker. The nurse explains this medication will block the renin-angiotensin-aldosterone system (RAAS) which, when activated, causes which assessment findings? SELECT ALL THAT APPLY.

-increase in peripheral vascular tone -renal retention of sodium and water

A client who has been awaiting the results of a bone marrow biopsy for several days is experiencing stress as a result of uncertainty and the possibility that abnormal cell growth may be detected. A physical examination and blood work would most likely yield which result?

-increased blood pressure -increased heart rate -increased antidiuretic hormone (ADH)

A client is concerned about acquiring hepatitis A and asks the nurse who should receive the HAV vaccine. Which response by the nurse is best? SELECT ALL THAT APPLY.

-international travelers visiting to regions where sanitation is poor -sexually active gay men -users of illicit drugs -workers employed in food handling

Which symptoms accompanying shortness of breath indicate a client has for pulmonale? SELECT ALL THAT APPLY.

-jugular vein distention -warm moist skin -2+ pitting edema in feet

The nurse is caring for a client with severe acute pancreatitis. Which interventions should the nurse prioritize ? SELECT ALL THAT APPLY.

-keep client NPO -infuse intravenous fluids -maintain nasogastric tube

The parents are ready to take home their child with newly diagnosed hemophilia A. Which teaching aspects should the nurse discuss with them prior to discharge? SELECT ALL THAT APPLY.

-keep the child away from contact sports like football and wrestling -administration of factor VIII at home when bleeding occurs

The nurse teaches a client with a new diagnosis of asthma about measure to prevent asthma attacks. What should the nurse include in the teaching plan? SELECT ALL THAT APPLY.

-limit exposure to dust -schedule the influenza vaccine -use an inhaled cromolyn

When caring for clients with alcoholism and cirrhosis, which of these does the nurse recognize are potential complications of the disease? SELECT ALL THAT APPLY.

-liver failure -bleeding esophageal varices -kidney failure

A client has been diagnosed with dysfunction of the anterior pituitary gland. The nurse is aware that which hormones will likely be affect? SELECT ALL THAT APPLY.

-lutenizing hormone (LH) -adrenocorticotropic hormone (ACTH) -thyroid-stimulating hormone (TSH) -growth hormone (GH)

The nurse is educating a client with hyperthyroidism who has recovered from thyrotoxicosis (thyroid storm) and is preparing for discharge. What can the nurse inform the client that this hypermetabolic state can be caused by in order to decrease exacerbation? SELECT ALL THAT APPLY.

-manipulation of the thyroid gland -physical or emotional trauma -stress

A motor vehicle accident has resulted in the driver hitting the steering wheel with subsequent fractures of the chest, sternum, and some ribs. Which manifestations would lead the nursing staff to suspect the driver has developed a tension pneumothorax? SELECT ALL THAT APPLY.

-mediastinal shift of the trachea toward one side -subcutaneous emphysema palpated in the upper chest/neck region

A single parent is grieving the recent loss of his only child and has limited coping abilities. Which health problem may be induced by this acute stress? SELECT ALL THAT APPLY.

-obesity -migraines -hypertension -anxiety

A client has been admitted with hepatocellular jaundice as a result of taking medications. From the following list of medications in the client's home, the nurse knows which medications may place the client at high risk for developing hepatocellular jaundice? SELECT ALL THAT APPLY.

-oral contraceptives for birth control -anabolic steroids illegally taken for body building -isoniazid for tuberculosis

A client with chronic hepatitis B is undergoing diagnostic testing for possible liver cancer. The nurse can correlate which clinical manifestations with development of liver cancer? SELECT ALL THAT APPLY.

-reports "abdominal fullness and bloating" -states "my abdomen is getting larger since and I can no longer get my belt buckled."

Which statement is true concerning dysplasia? SELECT ALL THAT APPLY.

-results in cell growth that produces a variety of specific tissue cell shapes. -is known to frequently exist in the respiratory tract. -is a strong precursor of cancer. -can result from chronic irritation.

A nurse is assessing a client for the classic signs of acute inflammation. The nurse would assess the client for:

-rubor -swelling -pain

The nurse is caring for a client diagnosed with hyperthyroidism. Which clinical manifestations are most likely to be seen in this client? SELECT ALL THAT APPLY.

-shortness of breath -tachycardia -weight loss -fine muscle tremor

Two years after chemotherapy and radiation therapy for lung cancer, a 72-year-old client reports being extremely tired all the time. The physician suspects the client may have developed aplastic anemia. The nurse assessing the client will likely find which clinical manifestation of aplastic anemia? SELECT ALL THAT APPLY.

-small spots of skin hemorrhages over entire body -complaints of weakness and fatigue -excess bleeding from gums and nose

A family member asks the nurse, "What do they mean when they start talking about stem cells?" What is the nurse's best response?

-stem cells are undifferentiated cells with the capacity to generate multiple different cell types.

The cause of gastric carcinomas has been influenced by which of the following factors? SELECT ALL THAT APPLY.

-strain of Helicobacter pylori (H. pylori) -environmental factors -family history

The nursing students have learned in class that causes of urinary obstruction and urinary incontinence include which of the following? SELECT ALL THAT APPLY.

-structural changes in bladder -structural changes in urethra -impairment of neurologic control of bladder function

While caring for a client undergoing diagnostic testing for possible liver failure, the nurse asks the student nurse what are clinical manifestations of liver failure they should asses for. Which answers are accurate? SELECT ALL THAT APPLY.

-sweet, musty breath -multiple bruising noted on body

The stress response involves the activation of which physiological systems? SELECT ALL THAT APPLY.

-sympathetic nervous system (SNS) -HPA axis -immune system

Which clients are at risk for impaired normal red blood cell (RBC) destruction? SELECT ALL THAT APPLY.

-the client diagnosed with chronic cirrhosis of the liver -the client whose spleen was damaged in an automobile accident -the client whose biopsy revealed cancer cells in numerous lymph nodes

Which clients are at risk for impaired normal red blood cells (RBC) destruction? SELECT ALL THAT APPLY.

-the client whose spleen was damaged in an automobile accident -the client diagnosed with chronic cirrhosis of the liver -the client whose biopsy revealed cancer cells in numerous lymph nodes

The nurse is caring for a client with sepsis who has developed disseminated intravascular coagulation (DIC). Which assessments should the nurse prioritize? SELECT ALL THAT APPLY.

-urine output -level of consciousness -oxygen saturation

A client with small cell lung cancer (SCLC) has developed a paraneoplastic syndrome called Cushing syndrome. Based on this new complication, the nurse will likely assess which clinical manifestations of Cushing syndrome?

-weight gain -moon face -buffalo hum, purple striae on abdomen

A client is admitted to the emergency department with possible internal bleeding after being involved in an automobile accident. What type of isotonic intravenous (IV) solution does the nurse prepare to infuse?

0.9% NaCl

An asymptomatic client who is worried about developing breast cancer due to the fact that is runs in her family asks the nurse if she could have a mammogram to see if she has any lumps. The nurse informs the client that a tumor usually is undetectable until it has doubled 30 times and contains more than 1 billion cells. This means that at this point it measures approximately which size?

1 cm

Oxygen has been prescribed for a client with chronic obstructive pulmonary disease (COPD). Which amount of oxygen is considered most appropriate for the COPD client?

1 to 2 L/min

A client with pneumonia is admitted with these vital signs: temperature 99.7 (37.6), pulse 80 beats/min, respirations 18/minute, and BP 120/80 mm Hg. Which set of vital signs does the nurse anticipate when the client begins to shiver and requests another blanket several hours later?

100.9 (38.3) respirations - 20/min Pulse - 90/min BP- 126/80 mm Hg

How many days do erythrocytes live in circulation?

120 days

The nurse is monitoring hourly urine output of a client diagnosed with hypovolemic shock. The nurse is most concerned if the client's output is:

20 mL/hour

A child diagnosed with Down syndrome has experienced a chromosomal abnormality involving which trisomy?

21

A client has just begun to experience an ischemic stroke. The blood supply fro the middle cerebral artery is being blocked by a large blood clot. How long before brain cells begin to die due to lack of ATP?

4 to 6 minutes

Which client is at high risk for developing dilated cardiomyopathy?

44-year-old noncompliant female who forgets to take her hypertensive medications.

The physician suspects a client may have Klinefelter syndrome. To confirm the diagnosis, the chromosome pattern would identify:

47, XXY

Body weight consists of which percentage of body water?

60%

After receiving change-of-shift report about the following four patients, which patient should the nurse assess first?

70-year-old returning from PACU following partial thyroidectomy who is extremely agitated, has an irregular pulse rate of 134, and an elevated temperature of 103.2.

The nurse has just received the lab results of a client's calcium level. The nurse identifies a normal calcium level as:

9.0 to 10.5 mg/dL (2.225 mmol/L to 2.63 mmol/L)

A client is admitted to the cardiac unit with a diagnosis of pericarditis. The nurse is teaching the client about the anatomical location of the infection. The nurse evaluates the effectiveness of the teaching when the client correctly identifies which of the following as the location of the pericardium?

A membranous sac that encloses the heart

The nurse is caring for four clients. Select the client who is at greatest risk for decreased wound healing.

An 80-year-old diabetic client with a foot ulcer.

Which statement is true regarding drug therapy and its effect on the body?

Antineoplastic cells directly damage cells.

A client comes to the clinic following exposure to chicken pox. The client states he had chickenpox as a child but was worried about getting sick again. Which statement best explains humoral-mediated immunity to the client?

B lymphocytes (B cells) in the body produce antibodies to fight infections. Once exposed, the cells retain memory and are able to quickly fight off pathogens during re-exposure and prevent the disease from reoccurring.

A teenage client has been diagnosed with infectious mononucleosis and asks the health care provider what caused the condition. Which response is most accurate for the nurse to share with this client?

Epstein-Barr virus (EBV)

Which of the following is the most common cause of lower uncomplicated urinary tract infections?

Escherichia coli

The nurse reviews the lab results of a client who has a thrombocyte count of 60 x 10 3 /uL (60x10 9/L). The client is at risk for:

Bleeding

The efficiency of the heart as a pump often is measured in the amount of blood the heart pumps per minute. which is the correct formula to figure out the cardiac output?

CO = SV x HR

The nurse recognizes the role of the lungs in acid-base balance is regulation of which of the following?

CO2

A nurse educator is explaining basic neuroanatomy to a class of prospective nursing students. Which statement best conveys an aspect of the role of cerebrospinal fluid (CSF)?

CSF cushions the brain and provides a near-water medium for diffusion of nutrients.

Which disorder is a result of excess cortisol?

Cushing syndrome

Which option most accurately describes the physiologic process resulting in more specialized cells with each mitotic division?

Differentiation

The nurse's hand-off reports states that the client has pyrexia. The nurse plans care for the client who has:

Fever

In the balance of secretions in the gastric mucosa by the parietal cells, which of the following ions is produced to buffer the production of hydrochloric acid?

HCO3-

When interpreting the arterial blood gas, the nurse recognizes which of these reflects the value for bicarbonate?

HCO3-

A client tells the nurse that he is concerned about developing hepatitis after being exposed to contaminated feces, saliva, and food. The nurse is aware that the client is a risk for:

Hepatitis A

Which enzyme has a powerful vasodilator effect on arterioles and increases capillary permeability?

Histamine

A lymph node biopsy pathology report notes the presence of Reed-Sternberg cells on a client suspected of having a lymphoma. The nurse interprets the report as indicating:

Hodgkin lymphoma

The nurse is caring for a client who survived a severe motor vehicle accident 2 months ago that killed her close friend. The client has expressed increased irritability and has started driving at least 15 miles/hour slower than the posted speed limit. Which stage of postraumatic stress disorder (PTSD) is she experiencing?

Hyperarousal

The nurse is caring for a client whose serum potassium level is 2.6 mEq/L (2.6 mmol/L). The nurse anticipates which intervention will be prescribed?

IV infusion of 10 mEq potassium chloride in 100-mL normal saline solution over 1 hour times three doses

A client presents to his physician complaining of severe fatigue and intermittent shortness of breath. The physician is concerned that the client developed anemia, a decreased oxygen carrying capacity of the blood, and has ordered blood work. A deficiency in which of the following would support the diagnosis of anemia secondary to a decrease in oxygen's ability to bind to hemoglobin?

Iron

The rate at which hemoglobin is synthesized depends on availability of which substance?

Iron

The nurse is educating a pregnant client about the importance of dietary folic acid to prevent anemia. Which food source will the nurse encourage because it is high in folic acid?

Kale

A client's bone marrow study report reveals the findings of blast cells in the bone marrow. What does the nurse interpret this as indicating?

Leukemia

The nurse is evaluating the bloodwork results of a client with an infected leg ulcer. The white blood cell count if 18,000 cells/uL. The nurse interprets this as:

Leukocytosis

The nurse has just completed teaching a client newly diagnosed with type 1 diabetes about rapid-acting insulin. The nurse determines that teaching was effective when the client selects:

Lispro

The nurse confers with the attending physician about a client with encephalopathy who has been attributed to mitochondrial gene mutation. Why do mutations of the mitochondria affect the brain?

Mitochondrial mutations affect the ability of tissues to synthesize ATP, and tissues with high demands malfunction without enough ATP.

The nurse is developing a community program since a recent increase in admissions to the acute care facility with tuberculosis infection. What is the most frequent form of tuberculosis that the nurse should focus on?

Mycobacterium tuberculosis

A nurse is caring for a client with a genetic disorder of mitochondrial DNA assesses for abnormalities in which body system?

Neuromuscular

A nurse sends a blood sample to the lab for analysis. Assuming the sample is normal, the nurse anticipates which white blood cells (WBCs) will account for the highest percentage?

Neutrophils

A nurse orienting to the surgical suite is studying medications that affect platelet function and notes that the most common medications are:

Nonsteroidal anti-inflammatory drugs (NSAIDS) and aspirin

What disease results from the degeneration of the dopamine nigrostriatal system of the basal ganglia?

Parkinson disease

The nurse is providing client education to a client newly diagnosed with Parkinson disease. The nurse most accurately describes the disease progression as:

Parkinson's is a chronic condition and treatment aims to manage symptoms

Which physiological process best describes the removal of microorganisms (such a bacteria), toxins, and cellular debris from the body?

Phagocytosis

A female client tells the health care provider that she has recently been experiencing episodes of changes occurring in the color of her fingertips, especially when she experiences cold temperatures. She further states that the tips become pale, turn a bluish color, and then become reddened. The client is most likely experiencing:

Raynaud phenomenon

a client in the acute stage of inflammation will experience vasodilation of the arterioles and congestion in the capillary beds. The nurse would assess the client's skin for:

Redness

A client with a known history of intravenous drug abuse has been diagnosed with infective endocarditis. Select the most likely cause of infection.

Staphylococcus aureus

A 42-year-old female client with breast cancer has a tumor that is minimal in size and extension, has minimal regional lymph nodes involved, and has no distant metastasis. Using the TNM system, the tumor is staged as:

T1, N1 and M0

A client arrives in the clinic and states to the nurse, "I am tired all the time and have gained weight. My hair is so dry it is breaking." The nurse assesses that the client's face is puffy with edematous eyelids and the outer third of the eyebrows are thinning. What lab test will the nurse prepare the client for that is characteristic of this disorder?

T4 and TSH

A nurse observes that a client's urine is cola colored and considers which factor as a possible reason?

The client's urine contains material from the degradation of red blood cells.

The nurse is caring for a 2-day-old newborn infant who appears lethargic and has a yellowish tint to the skin. Select the most likely cause of this newborn's signs and symptoms.

The inability of the immature liver to conjugate bilirubin

T or F: Bruising of the skin is a manifestation of bleeding disorders in patients with CKD.

True

The provider is doing genetic counseling with a client. He explains that which condition affects only females?

Turner syndrome

Select the statement that best describes stem cells.

Undifferentiated cells of continuously diving tissues that have the capacity to generate multiple cell types.

The nurse is assigned to care for four clients on a medical floor. Which client is most likely at risk for viral pneumonia after influenza?

a 76-year-old client with cardiopulmonary disease

What is the correct definition to describe a carrier?

a carrier is heterozygous for a recessive trait but does not manifest the trait.

Which person should the charge nurse assign as a roommate for a client diagnosed with aplastic anemia?

a client in traction with a broken femur

The nurse is reviewing the medical history of four clients. The nurse determines that the client at greatest risk for developing peptic ulcer disease is:

a client with a prior diagnosis of Helicobacter pylori and refused treatment

A client is admitted for a relapse for sarcoidosis. Knowing this is usually caused by an inflammatory process, the nurse can anticipate administering:

a corticosteroid

A client is diagnosed with hyperthyroidism and is exhibiting weight loss, diarrhea, and tachycardia. What does the nurse understand that these clinical manifestations are related to?

a hypermetabolic state

A client has been diagnosed with an abscess. Upon assessment of the client, the nurse would expect to find:

a localized pocket of infection composed of devitalized tissue, microorganisms, and the host's phagocytic white blood cells.

A nurse is evaluating clients for the risk of developing type 2 diabetes. Which client has the highest risk for developing this metabolic disorder?

a middle aged obese adult with sedentary lifestyle

the client tells the provider he has lower back pain. Although most abdominal aneurysms are asymptomatic, the provider examines the client for which initial sign of this condition?

a pulsating mass in the abdomen

The nurse observes a client care technician obtain a blood pressure on an obese client using a BP cuff that is too small. The nurse is aware that this will result in:

a reading that is too high

A client recovered from influenza 2 days ago and informs the nurse that she is feeling better but how has a fever, chills, pain when breathing, and a productive cough. What complications does the nurse anticipate the client will be treated for?

a secondary bacterial pneumonia

The nurse and nursing student are caring for a client undergoing a severe stressor with release of epinephrine into the bloodstream. Which of these effects on blood glucose levels does the nurse teach the student epinephrine will cause?

blood glucose will elevate

A 26-year-old female is resting after a 1-minute episode during which she lost consciousness while her muscles contracted and extremities extended. This was followed by rhythmic contraction and relaxation of her extremities. On regaining consciousness, she found herself to have been incontinent of urine. What has the woman most likely experienced?

a tonic-clonic seizure

A patient exhibiting problems with their thyroid has been scheduled for a radioactive scan. From the following list of patients, which would the nurse question as to whether those would be a safe procedure for this patient?

a young female patient who has been trying to get pregnant.

Which symptoms is often observed in cases of peritonitis?

abdominal rigidity

Sometimes the host's white blood cells are unable to eliminate the microorganism, but the body is able to contain the dissemination of the pathogen. What is this called?

abscess

The nurse evaluating a client's blood work determines that a client has an increased risk for infection based on which lab results?

absolute neutrophil count of 800/uL (0.80 x 10 9/L)

The nurse is assessing a male client and finds abnormally large hands and feet, a bulbous nose, and a broad face with a protruding jaw. Based on these findings, which endocrine abnormality is most likely the cause for these physical changes?

acromegaly

What is the name of the process during which cells use energy to move ions against an electrical or chemical gradient?

active transport

Which client is displaying manifestations of having a kidney stone?

acute onset of colicky flank pain radiating to lower abdomen

A client is brought to the emergency department and is diagnosed with an ischemic stroke confirmed by CT scan. The most important treatment for this client would be to:

administer IV tissue plasminogen activator (tPA)

The critical care nurse has just admitted a client with diabetic ketoacidosis (DKA) whose blood glucose level is 877 mg/dL. The client's breath has a fruity odor and the client is confused. Which of these does the nurse set as the priority at this time?

administration of intravenous fluids

The nurse is caring for a client who has developed hypoxemia and tissue hypoxia. Which of these interventions does the nurse set as a priority intervention?

administration of supplemental oxygen

The nurse is caring for inmates in a correctional facility. Which is the mode of transmit for the recent outbreak of tuberculosis?

airborne droplets

According to Hans Selye, which is the correct order for the stages of general adaptation syndrome?

alarm resistance exhaustion

Which plasma proteins is considered the most abundant type?

albumin

The mother is heterozygous for blue eyes, a recessive trait. The father is homozygous for brown eyes, a dominant trait. What color eyes will their four children have?

all will have brown eyes

Global and focal brain injuries manifest differently. What is almost always a manifestation of a global brain injury?

altered level of consciousness

Anticoagulant drugs prevent thromboembolic disorders. How does warfarin, one of the anticoagulant drugs, act on the body?

alters vitamin K, reducing its ability to participate in the coagulation of the blood.

The nurse is caring for a client who is now 2 days post near-drowning. The focused assessment would involve which are of the lung involved in gas exchange?

alveoli

When assessing the client with acute pancreatitis, which of these diagnostic tests, consistent with the disease, does the nurse anticipate will be altered?

amylase and lipase

The criteria for fetal alcohol syndrome diagnosis require the documented presence of all of the following except which one?

an absence of all of part of the X chromosome

A client has just been diagnosed with multiple sclerosis (MS). The nurse recognizes that the client's condition is a result of:

an immune-mediated response that is caused by the demyelinization of the myelin sheath of the white matter o the brain, spinal cord, and optic nerve.

A 20-year-old male client is experiencing a severe immunologically mediated reaction, in which histamines have been released into the blood. Which type of reaction is most likely occurring with this client?

anaphylactic shock

A client has developed a tumor of the posterior pituitary gland. The client is at risk for problems with secretions of:

antidiuretic hormone (ADH) and oxytocin

A nurse is teaching a client newly diagnosed with a seizure disorder. Which statement is most important for the nurse to provide regarding anti epileptic medications?

antiepileptic medications should never be discontinued abruptly

Normal physiologic process involves the necessary removal of irreversibly damaged cells. Which term best describes this process?

apoptosis

A client is suffering from severe gastroesophageal reflux disease (GERD) and has been admitted to the hospital with a diagnosis of pneumonia. Which of the following would be the most likely cause for the development of pneumonia?

aspiration

An older adult client recently had a cerebrovascular accident that resulted in dysphagia. What is the nurse's greatest concern while feeding this client?

aspiration

How does low-dose aspirin therapy impact clot formation?

aspirin is a platelet aggregation inhibitor.

The nurse is caring for a client with a brain tumor when the client beings to vomit. Which intervention should the nurse do first?

assess for other signs/symptoms of increased intracranial pressure

A client has sudden severe dyspnea, fear, asymmetrical chest movement and decreased lung sounds on the right side. Which intervention is most appropriate?

assist to high-Fowler's position and prepare for chest tube insertion.

Prolonged immobility is implicated in the development of which disorder?

atelectasis and pulmonary embolism

Which substance released by atrial muscle cells will inhibit sodium and water reabsorption?

atrial natriuretic peptide (ANP)

The parasympathetic nervous system is part of which system?

autonomic nervous system

Select the response that best describes the pressure-sensitive receptors that respond to changes in the stretch of the vessel wall.

baroreceptors

A client with a 25-year history of smoking is diagnosed with emphysema. Physical assessment reveals an increased anterior-posterior chest diameter. Which term should the nurse use to document this finding?

barrel chest

The nurse is caring for a client who has had acute blood loss from ruptured esophageal varices. What does the nurse recognize is an early sign of prerenal failure?

baseline urine output of 50 mL/hr that is now 10 mL/hr

A client's echocardiogram identified a narrowed valve that has resulted in a decreased blood flow between the left atria and left ventricle. The nurse would interpret this as the:

bicuspid valve

Following a collision while mountain biking, the diagnostic work up of a 22-year-old male has indicated the presence of an acute subdural hematoma. Which pathophysiologic process most likely underlies his diagnosis?

blood has accumulated between the man's dura and subarachnoid space

A nurse is caring for a client with systemic lupus erythematosis (SLE). The client asks why a urinalysis is necessary. The best answer is that a urinalysis would determine whether which factor was present in the urine?

blood or protein

Select the option that best describes the production of T lymphocytes.

bone marrow - thymus - lymph nodes

A nursing student has learned in class that with abdominal cancer, the tumor may compress the viscera, which will cause which of the following?

bowl obstruction

When caring for the client with diabetic ketoacidosis, the nurse recognizes that fatty acids and ketones may be used for energy by most organs. Which of these organs does the nurse recognize is reliant on glucose as the major energy source?

brain

Manifestations of brain tumors are focal disturbances in brain function and increased ICP. What causes the focal disturbances manifested by brain tumors?

brain edema and disturbances in blood flow

The health care provider is teaching a client about the metabolic effect of cortisol. The most appropriate information to provide would be:

breakdown of proteins and fats

A client experiencing immotile cilia syndrome (Kartagener syndrome) should be frequently assessed by the nurse for which priority complication?

bronchiectasis due to interferences with clearance of inhaled bacteria along the respiratory tract.

A client is being treated for stomach cancer. The client is in considerable and constant pain, and the family is asking why. How does soft tissue cancer cause pain?

by compressing and eroding blood vessels, causing ulceration and necrosis, along with frank bleeding and sometimes hemorrhage.

The nurse in the oncology unit has just admitted a client with metastatic cancer. The client asks how cancer moves from one place to another in the body. What would the nurse answer?

cancer cells enter the body's lymph system and thereby spread to other parts of the body.

Select the correct sequence of blood return to the heart.

capillaries, venules, veins, right atrium

Which complication of acromegaly can be life threatening?

cardiac structures increase in size

The nurse is caring for a client who has just experienced an acute myocardial infarction. Which type of shock is this client likely to experience?

cardiogenic

Marfan syndrome, which is a connective tissue disorder, affects several organ systems. The most life-threatening aspects of the disorder are caused by defects in which system?

cardiovascular

When caring for a client with hyperkalemia, the nurse priorities assessment of which body system?

cardiovascular

Which is considered a function of plasma?

carrying nutrients

A client who has been experiencing tremendous stress has developed pneumonia, which he ultimately attributes to the stressors he has been facing. This phenomenon may result from:

catecholamine release

What is the common gram-negative bladder infection found in hospitalized clients?

catheter-induced infection

A client with gastroesophageal reflux disease has metaplasia. Which explanation is the cause?

cells are replaced in response to chronic irritation.

A 20-year-old female comes to the clinic for an annual exam. She reports having unprotected sexual intercourse with several men over the last year. Blood tests reveal that she is positive for human papillomavirus. The nurse teaches the patient about which of the following complications that may occur because of the infection?

cervical cancer

The client with diabetes mellitus has fatigue, nauseas, a fruity odor to the breath, and polyuria. Which should the nurse do first?

check blood glucose

The nurse determines that the client has clubbing of the fingertips. Which is the best intervention?

check the client's O2 saturation level.

The family of an older adult reports increasing inability to perform basic activities of daily living. After evaluation, the client is diagnosed with Alzheimer's disease. What intervention will be implemented to slow cognitive decline?

cholinesterase inhibitors

A woman gives birth to a small infant with a malformed skull. The infant grows abnormally slowly and shows signs of substantial cognitive and intellectual deficits. The child also has facial abnormalities that become more striking as it develops. What might you expect to find in the mother's pregnancy history?

chronic alcohol use

The physician is examining a postmenopausal female and explains that which cause of iron deficiency is most common in adults?

chronic blood loss

An older adult client asks the nurse why so many older people develop anemia. Which response by the nurse is most accurate?

chronic disease

Which individual is at greatest risk for developing a venous thrombosis resulting from venous stasis?

client on bed rest

When assessing the nutritional intake of a client with generalized pancreatic cancer, which of these statements by the client is consistent with the disease and food intake?

client states pain becomes worse with food intake

The nurse is caring for four male clients recognizes which client is at higher risk for developing post renal kidney failure?

client with prostatic hyperplasia

For clients with heart failure, what pathophysiologic response helps maintain the cardiac reserve?

compensatory mechanisms

If a client experiences sympathetic nervous stimulation of the heart, the nurse will observe which changes in manifestations?

increased heart rate and increased contractility

Which of the following criteria about insulin would prompt a diagnosis of type 1 diabetes?

complete failure of insulin secretion

Abnormalities of body structure, function, or metabolism that are present at birth are known as:

congenital

A client has developed chronic hypoxia and has developed pulmonary hypertension (HTN). The nurse recognizes that the most likely cause of pulmonary hypertension would be:

constriction of the pulmonary vessels in response to hypoxemia.

The nurse is preparing to administer a pregnant client's medication when it is noted that the client is prescribed tetracycline for an infection. The nurse should do which of the following?

contact the physician

The nursing student was initially overwhelmed with the demands of nursing school. The student implemented a plan of getting adequate sleep and nutrition to adapt to the new demands. These strategies are known as which of the following?

coping mechanisms

A client with primary lung disease has developed right heart failure. The health care provider would document this as:

cor pulmonale

A client is diagnosed with adrenocorticotropic hormone deficiency (ACTH) and is to begin replacement therapy. Regarding which type of replacement will the nurse educate the client?

cortisol replacement therapy.

Which manifestation of left-sided heart failure can be diagnosed by examination of the lips and mucous membranes?

cyanosis

When explaining to a client why there was only minimal muscle damage following 99% occlusion of the left anterior descending artery, the nurse will explain that this is primarily due to:

development of collateral circulation that build channels between some of the smaller arteries, usually when the flow is decreased gradually.

Unlike other teratogens, alcohol exposure during pregnancy (fetal alcohol spectrum disorders) can have what harmful effects on the fetus?

developmental abnormalities throughout the prenatal and postnatal period.

During the admission interview the client, who is admitted with bacterial pneumonia, reveals a 20 pack per year smoking history. The nurse relates the possible cause of this pneumonia to the decreased defense of the pulmonary system cause by cigarette smoking. Smoking affects the pulmonary defense system in what way?

damage or destruction of cilia

The nurse is caring for a 31-year-old trauma victim admitted to the neurologic intensive care unit. While doing the initial assessment, the nurse finds that the client is flexing the arms, wrists, and fingers. There is adduction of the upper extremities with internal rotation and plantar flexion of the lower extremities. How would the nurse describe this in the notes?

decorticate posturing

The parents of an infant born with hydrocephalus are concerned about the size of the baby's head. The doctors are telling them that the infant needs the surgical placement of a shunt. The nurse caring for the infant in the neonatal intensive care unit explains that placement of a shunt will:

decrease the likelihood of further neurological deficits.

A client has been prescribed a thiazide diuretic, hydrochlorothiazide (HCTZ), for the initial treatment of hypertension. What effect does the nurse know this drug will have on the body to consequently decrease blood pressure?

decrease vascular volume

A nurse in the emergency department admits a male client who has experienced severe frostbite to his hands and toes after becoming lost on a ski hill. The nurse recognizes that which phenomena has contributed to his tissue damage?

decreased blood flow has induced hypoxia.

The nurse is caring for a client who has a low level of T lymphocytes. The nurse plans care for a client with:

decreased immune response

The nurse reading a client's history and physical notes that the client has a history of leukopenia. The nurse interprets this to mean that the client has:

decreased leukocytes

A client who experienced an ST elevation myocardial infarction (STEMI) received fibrinolytic therapy with streptokinase.Which manifestation alerts the nurse to a developing complication?

decreased level of consciousness

The health care provider is concerned that a client may be at risk for problems with cerebral blood flow. The most important data to assess would be:

decreased level of oxygen

A client experiences an increase in thyroid hormone as a result of a thyroid tumor. Which hormonal response demonstrates the negative feedback mechanism?

decreased thyroid-stimulating hormone (TSH)

The client has been diagnosed with impaired blood flow in the deep venous channels of her legs. The nurse explains that the most common cause of this condition is:

deep vein thrombosis (DVT)

The nurse is explains to the parents of a 23-week premature infant the reason their baby needs to be on mechanical ventilation. The education is successful when the parents state that the understand their baby was born before the type II alveolar cells could mature, which has caused a:

deficiency of surfactant

A woman reports to the nurse that she has developed a yeast infection. The woman does not understand how she could get a yeast infection since she has been on antibiotics for a urinary tract infection. What is the rationale for the client's complaint?

destroying one type of resident flora (bacteria) can allow overproliferation of another competing type (yeast).

Your client with end-stage renal disease is receiving 2 units of packed red blood cells for anemia (Hgb of 8.2 g/dL 982 g/L)). Twenty minutes into the first transfusion, the nurse observes the client has a flushed face, hives over upper body trunk, and is reporting pain in lower back. His vital signs include pulse rate of 110 and BP drop to 95/56. What is the nurse's priority action?

discontinue the transfusion and begin an infusion of normal saline

Your client with end-stage renal disease is receiving 2 units of packed red blood cells for anemia (Hgb of 8.2 g/dL(82g/L)). Twenty minutes into the first transfusion, the nurse observes the client has a flushed face, hives over upper body trunk, and is reporting pain in lower back. His vital signs include pulse rate of 110 and BP drop to 95/56. What is the nurse's priority action?

discontinue the transfusion and begin an infusion of normal saline

An older adult client is diagnosed with orthostatic hypotension. For which clinical manifestation should the nurse assess?

dizziness and fainting

A client affected by postural hypotension will likely display what symptoms?

dizziness and pallor when moved to upright position

A client is admitted to the hospital with a diagnosis of deep vein thrombosis and started on intravenous heparin therapy. Seven days later, the client's lab values identify a rapid decrease in platelets. The health care provider recognizes this as:

drug-induced thrombocytopenia

A client with pancreatitis is admitted with weight loss, nausea, and vomiting. To maintain nutrition, the physician orders parental nutrition to be started. Knowing that a major side effect of parental nutrition is a hyperosmolar hyperglycemic state, the nurse should assess the patient for which clinical manifestations (listed below)?

dry lips, excess urine output, and seizures

Which of the meninges provides the major protection for the brain and spinal cord?

dura mater

A client who has no previous history of respiratory disease describes a sensation of shortness of breath and the feeling of not being able to pass enough air during exercise that has just developed over the past month. The client would be documented as having:

dyspnea

Which laboratory findings should you expect to see in a patient diagnosed with nephritic syndrome?

elevated urine protein and hypoalbuminemia

An older male comes to the clinic with the chief complaint of having difficulty voiding. The physician diagnoses him with a lower urinary tract obstruction and stasis. Which of the following should the nurse suspect to be the most frequent cause of this client's problem?

enlargement of prostate gland

When caring for a client with anemia and a decrease in red blood cells (RBCs), the nurse recognizes which of these hormones will stimulate the bone marrow to produce additional RBCs?

erythropoietin

When can a teratogenic environmental agent cause birth defects?

exposure during early pregnancy

Bradykinesia occurring in Parkinson disease places the Parkinson's client most at risk for:

falls and injury

The nurse is caring for a client with a tumor obstructing the lymphatic system. For which consequence does the nurse assess?

fluid accumulating in the interstitial spaces distal to the tumor

In patients with acute diarrhea, many require no treatment. However, the nurse knows the assessment in all patients with diarrhea is which of the following?

fluid and electrolytes

The emergency room provider diagnoses a client with a hemothorax. Which could be possible causes of this condition?

fractured ribs following car accident

The nurse is caring for a client who received regular insulin at 7 am. Four hours later the nurse finds the client diaphoretic, cool, and clammy. Which of these interventions is the priority?

give the client a concentrated carbohydrate

The nurse measure a blood glucose level of 40 mg/dL (2.22 mmol/L) for a client with type 1 diabetes. Why would it be important for the nurse to institute an intervention to elevate the glucose level in this client?

glucose is not stored in the brain and is a major fuel source for brain function

a 25-year-old female client exhibits exophthalmos of both eyes. The health care provider recognizes this as a manifestation of:

graves disease

A nurse will be providing care for a female client who has a diagnosis of heart failure that has been characterized as being primarily right-sided. Which statement best describes the presentation that the nurse should anticipate? The client:

has pitting edema to the ankles and feet bilaterally, decreased activity tolerance, and occasional upper right quadrant pain.

The nurse is caring for a client whose temperature is increasing. Which other vital sign/body response will also increase during fever?

heart rate

The nurse is performing an assessment for a client who has hyperthyroidism that is untreated. When obtaining vital signs, what is the expected finding?

heart rate 110 and bounding

A client is diagnosed with leukemia. The student nurse working with this client correctly identifies this type of cancer to be:

hematologic cancer

The nurse is caring for a client with diabetes. Which of these findings is cause for concern leading the nurse to initiate client education?

hemoglobin A1 C level is 8.7

A 29-year-old female has been admitted to the emergency department following a suicide attempt by overdose of acetaminophen. What physiologic changes in the client's liver causing which of these altered diagnostic results does the nurse anticipate?

hepatocellular necrosis evidenced by increased ALT and AST levels.

Each cell has 46 chromosomes, 23 from the mother and 23 from the father. The chromosomes are paired with alleles of genes that are paired, and if both members of the gene pair are identical, the person is considered homozygous. What is the person considered if both members of the gene pair are not identical?

heterozygous

Which enzyme has a powerful vasodilator effect on arterioles and increases capillary permeability?

histamine

The health care provider is discussing major risk factors for coronary artery disease (CAD) with a client. The most important information for the provider to include would be:

history of cigarette smoking and elevated blood pressure

The nurse explains to prospective parents that AA or aa in a Punnett square represent two alleles of a given pair that are the same, or:

homozygotes.

Which statement best explains the function of hormone receptors?

hormones receptors recognize a specific hormone and translate the hormonal signal into a cellular response.

A nurse is caring for a client who has a recent history of passing calcium urinary stones. Which of the following is a priority nursing consideration for this client?

hydration

The nurse is teaching a client diagnosed with Addison disease about the importance of lifetime oral replacement therapy. Which pharmacologic agent would be the drug of choice and included in this teaching plan?

hydrocortisone

A client asks why her breasts are enlarged during pregnancy. Which explanation is the best answer?

hyperplasia due to estrogen stimulation

What is the most common cause of drug fever?

hypersensitivity reaction to medication

A nurse is assessing a client for early manifestations of chronic kidney disease (CKD). Which would the nurse expect the client to display?

hypertension

The health care provider is teaching a client about modifiable risk factors for atherosclerosis. The most appropriate information to provide would be:

hypertension

Increased cardiac workload with left-sided heart failure can result in which change to the myocardial cells?

hypertrophy

Respiratory alkalosis can be caused by a respiratory rate in excess of that which maintains normal plasma PCO2 levels. What is a common cause of respiratory alkalosis?

hyperventilation

A health care provider is assessing a client for a potential endocrine disorder. Assessment findings identify abnormalities with emotion, pain, and body temperature. Which mechanism of endocrine control will require further laboratory/diagnostic assessment?

hypothalamus

Maintenance of blood gas concentrations, water balance, and food consumption are controlled by which part of the brain?

hypothalamus

While rock climbing, a 22-year-old male has endured a severe head injury. Which statement best captures expected clinical manifestations and treatments for his immediate condition?

hypoventilation may exist, resulting in increased PCO2 and hypoxemia that may require mechanical ventilation.

A 20-year-old college student has a pelvic fracture and a severed leg from a motorcycle accident. She lost several units of blood. When the student arrived in the emergency department, her blood pressure was very low, her pulse was high, and her skin was pale. The nurse knows that this client has developed which type of shock?

hypovolemic

A client diagnosed with congestive heart failure has an arterial blood oxygen level (PaO2) of 89%. Which term should the nurse use to document a reduced oxygenation of arterial blood?

hypoxemia

The nurse is caring for a client diagnosed with pneumonia. The client's arterial blood gas results identify decreased level oxygen and other laboratory work reveals an increase in lactic levels. How will the nurse interpret these findings?

hypoxemia

A client is admitted with an alteration in arterial blood gases. Cellular injury is most likely to result from which aspect of this abnormality?

hypoxia

A nurse is monitoring a client with anemia and low oxygen levels. The nurse knows that which condition stimulates the secretion of erythropoietin?

hypoxia

While being on subcutaneous heparin injections for deep vein thrombosis during the third trimester of her pregnancy, a client begins to experience major side effects. Her OB physician has called in a specialist who thinks the client is experiencing heparin-induced thrombocytopenia. The nurse should anticipate which order?

immediately discontinue the heparin therapy

Which function is the main job of lymphocytes?

immune reaction

The patient who has been admitted with a problem with his bladder has a postvoid residual (PVR) of 250 ml. The nurse understands that this indicates which of the following?

inadequate bladder emptying

A client has developed global ischemia of the brain. The nurse determines this is:

inadequate to meet the metabolic needs of the entire brain

A client is brought into the emergency department with severe crushing injuries to the chest wall and signs of respiratory failure following a motor vehicle accident. Which laboratory value would be expected?

increase in PCO2

When the nurse is performing a health history for a client who is being admitted for hyperthyroidism, what symptoms does the client report that the nurse would find associated with this disorder?

increase in appetite

A client is experiencing the early stages of an inflammatory process and develops leukocytosis. The nurse recognizes this as a/an:

increase in circulating neutrophils

Ventilation is driven by while alteration in arterial blood?

increased PCO2

The client is receiving chronic glucocorticoid therapy for chronic obstructive pulmonary disease (COPD). Which lab results would the provider expect to see?

increased blood glucose

Which assessment findings would the health care provider consider as most indicative of acute renal failure?

increased nitrogenous waste levels; decreased glomerular filtration rate (GFR)

The nurse is reviewing laboratory results of a client who has liver failure. Which finding would place the client at increased risk for bleeding?

increased prothrombin time

A nurse is teaching a client about the effects of UV radiation. Which risk factor is greatest following exposure to UV radiation?

increased risk of cancer

A child has developed respiratory stridor and is displaying a crowing sound. The parents ask the nurse what is causing this sound. The best response would be:

increased turbulence of air moving through the obstructed airways .

The nursing student studying about cancer growth correctly identifies "cell proliferation" to mean which of the following?

increasing cell numbers by mitotic cell division.

A client is to receive a radio contrast media as part of a diagnostic scan. Which intervention is intended to reduce the nephrotoxic effects of the radio contrast media?

increasing the normal saline intravenous infusion rate prior to the exam.

During the assessment of a client with heart failure, the nurse uses finger pressure to determine if edema is present in the lower extremities. When would the nurse document pitting edema?

indentation remains after the finger has been removed.

The nurse notes that the client has a decreased neutrophil count. The nurse recognizes that the client is at risk for which of the following?

infection

A client has just been told that he has an infection of the inner surface of the heart. He is also told that bacteria has invaded his heart valves. What term is used for this disease process?

infective endocarditis

Crohn's disease is treated by several measures. Treatment with sulfasalazine will focus on which aspect of this disease?

inflammatory suppression

A nurse is teaching a client with newly diagnosed hypertension about antihypertensive drug therapy. The nurse determines that the knowledge is understood when the client correctly describes which of the following as the mechanism of action of an angiotensin-converting enzyme (ACE) inhibitor?

inhibits the conversion of angiotensin I to angiotensin II, decreasing angiotensin II levels and reducing its effect on vasoconstriction

Select the most appropriate intervention for the nurse to teach a client diagnosed with distal symmetric neuropathy related to diabetes.

inspect the feet for blisters daily.

A nurse is assessing a 1-year-old child diagnosed with croup. Which manifestation would most likely be present?

inspiratory stridor and barking cough

The nurse is assessing a client diagnosed with anemia and notes that the client's skin and mucous membranes are pale. The nurse interprets this as:

insufficient hemoglobin

The diagnosis of type 1 diabetes would be confirmed by:

insulin is not available for use by the body.

The nurse is caring for a client who has been on an oral corticosteroid for several years for an autoimmune condition. The nurse notes several bruises of different stages of resolution on the client. What is the nurse's best response to this finding?

interview the client regarding side effects of corticosteroid therapy

The client reports something "wrong" with the gallbladder. Which manifestations does the nurse recognize that supports he client's concern?

intolerance to greasy food; burping

A female client comes to the clinic with symptoms of fatigue and heavy menses over the last 6 months. Laboratory tests reveal a microcytic hemochromic anemia. Based on these results, the nurse anticipates teaching the client about which type of anemia?

iron deficiency anemia

A client was recently diagnosed with iron deficiency anemia. The nurse anticipates which treatment to be prescribed?

iron supplementation and dietary teaching

A client who has undergone radiation therapy is expected to have some necrosis of cells. Necrosis is which type of side effect?

irreversible cell damage

A client has been diagnosed with cancer that was a result of dysfunctional apoptosis. The health care provider explains that the process to the multidisciplinary client are team. Select the best explanation.

it allows for DNA-damaged cells to survive.

Which statement describes the greatest diagnostic limitation of an electrocardiogram (ECG)?

it documents only current cardiac function

While discussing the heart, the nursing instructor teaches about pericarditis. Which statement does the instructor state best defines this disease?

it is an inflammatory process

While discussing the heart, the nursing instructor teaches about pericarditis. Which statement does the instructor state best defines this disease?

it is an inflammatory process.

A nurse is giving discharge information for a client who was diagnosed with acute pyelonephritis days previously. Which instruction is important for the nurse to discuss when teaching the client?

it is important that the client take the prescribed antibiotic for the duration of the prescription.

Genetic testing has revealed that a male infant has been born with an extra X chromosome. What are the most likely implications of this finding for the child?

it is likely to have no manifestations of this chromosomal abnormality.

A client has there arterial blood gas values: anion gap 20 mEq/L (20 mmol/L), pH 7.29, PCO2 37 mm Hg (4.92 mmol.L), HCO3- 11 mEq/L (11 mmol/L), base excess -6mEq/L (-6 mmol/L). With what condition do these values correspond?

lactic acidosis

The nurse is providing dietary instruction for a client with chronic kidney disease (CKD) which is on hemodialysis. Which food would the nurse encourage the client to restrict?

lean meats

The heart is a four-chambered pump. Which chamber of the heart pumps blood into the systemic circulation?

left ventricle

The nurse is reviewing the admission assessment of a client admitted with jaundice and abdominal pain. The client tells the nurse that he injured his back about 6 weeks ago and has been taking 750 mg of acetaminophen every 4 hours each day for pain relief. The nurse recognizes that client may have sustained damage to the:

liver

In some cancers, the presenting factor is an effusion, or fluid, in the pleural, pericardial, or peritoneal spaces. Research has found that almost 50% of undiagnosed effusions in people not known to have cancer turn out to be malignant. Which cancers are often found because of effusions?

lung and ovarian cancers

The nurse is conducting education for a client who is scheduled to undergo diagnostic testing for non-hodgkin lymphoma (NHL). The nurse includes information on:

lymph node biopsy

A nursing student is studying the respiratory airways of the lungs, in particular the alveoli. A fellow student asks which cells are most instrumental in the destruction of foreign substances that may enter the alveoli with inspired air. The most accurate response would be:

macrophages

A nurse caring for a client in myasthenic crisis identifies a priority concern as:

maintenance of airway and respiration

The nurse is aware that the primary function of the sympathetic nervous system is:

maintenance of vital functions and responding when there is a critical threat to the integrity of the individual.

The nurse is caring for an infant born with a cleft lip and palate. The priority of care would address:

malnutrition

A 40-year-old client is admitted to the hospital after experiencing 3 days of extreme vomiting. The doctor reviews the lab results and notes that the hemoglobin and hematocrit are elevated. What is the likely cause of these abnormal lab findings?

manifestation of dehydration

The nurse is reviewing the following lab results of a client diagnosed with renal failure: pH: 7.24 PCO2: 38 mm Hg (5.05 kPa) HCO3: 18 mEq/L (18 mmol/L) The nurse would interpret this as:

metabolic acidosis

A client tells the nurse that the client has been taking Alka-Seltzer (bicarbonate-antacid) four times a day for the past 2 weeks for an upset stomach. Upon assessment of the client, the nurse notes hyperactive reflexes, tetany, and mental confusion. Arterial blood gases reveal oH 7.55; serum HCO3- 37 mEq/L (37 mmol/L). The nurse suspects the client may be experiencing:

metabolic alkalosis

A client has developed a large wound as a result of trauma sustained during an auto accident. The cells will undergo which process to regenerate tissue?

mitosis

A biochemical event (e.g., nucleotide change, deletion, or insertion) that produces a new allele is called:

mutation

The nursing instructor is explaining the role of vascular smooth muscle cells in relation to increases in systemic circulation. During discussion, which neurotransmitter is primarily responsible for contraction of the entire muscle cell layer, thus resulting in decreased vessel lumen radius?

norepinephrine

A client diagnosed with Parkinson's disease is displaying the following manifestations: tremors, rigidity, and slowness of movement. The nurse would interpret these as:

normal manifestations of Parkinson disease

A nurse is explaining genetics to a client with an inherited disease. Which principle explains the fact that the client is very sick, yet his brother is not?

not all people with a mutant gene are affected to the same extent due to expressivity.

A client who developed a deep vein thrombosis during a prolonged period of best rest has deteriorated as the clot has dislodged, resulting in a pulmonary embolism. Which type of shock is this client at risk of experiencing?

obstructive shock

A client with a long history of stable angina suddenly experiences substernal pain that radiates to the left arm, neck, and jaw. He describes the pain as severe and feels as if he is suffocating. He has taken nitroglycerin and not experienced any relief. The client is most likely experiencing:

onset of STEMI

A client with severe hypoglycemia is unconscious. Which method of providing glucose should be avoided?

orange juice orally

Pressure generated as water moves across a membrane is also known as which of the following?

osmotic pressure

The nurse is reviewing a client's arterial blood gas (ABG) results. Which pH level would the nurse interpret as normal?

pH of 7.35-7.45

the nurse is reviewing a client's arterial blood gas (ABG) results. Which pH level would the nurse interpret as normal?

pH of 7.35-7.45

A patient is concerned about the possibility of having bladder cancer after his brother was diagnosed with it 2 years ago. Which of the following assessment data obtained by the nurse would indicate that the patient should be screened for this disease?

patient reports that he occasionally has blood in his urine but has no pain with it.

Peptic ulcer can affect one or all layers of the stomach or duodenum. Ulcers that penetrate through the muscularis layer are classified as which of the following types of ulcers?

perforation

A 75-year-old client with a history of heart valve replacement arrives at the outpatient clinic with multiple red pinpoint lesions. The nurse identifies the lesions as:

petechiae

The nurse is caring for a client with leukemia who is having difficulty with blood clotting after having several transfusions with packed red blood cells. What does the nurse anticipate infusing for this client to assist with controlling the bleeding?

platelets

Which condition/disorder would the nurse see as being likely to cause the most serious long-term problems?

polycystic kidney disease

A client tells the nurse that the doctor told her she has too many red blood cells accompanied by elevated white cells and platelet counts. The nurse recognizes this as:

polycythemia vera

The nurse is preparing a client with suspected leukemia for a bone marrow and biopsy. What preferred site will the nurse be sure is accessible for the physician?

posterior iliac crest

A client in renal failure has marked decrease in renal blood flow cause by hypovolemia, the result of gastrointestinal bleeding. The nurse is aware that this form of renal failure can be reversed if bleeding is under control. Which form of acute renal injury does this client have?

prerenal failure

A client is admitted with a suspected diagnosis of Hodgkin lymphoma (HL). Which diagnostic test would the nurse look for to confirm the diagnosis?

presence of Reed-Sternberg cells

An older male client has fallen at home and sustained a bruised left hip. Assessment reveals shortness of breath and shallow respirations with a respiratory rate of 40 breaths/min. If the client develops a tension pneumothorax, what additional assessment finding would the nurse observe?

presence of a mediastinal shift

The nurse is assessing the wound of a postoperative client. The client has a 6-inch abdominal would that is well approximated and closed with surgical suture. The wound does not display any redness or drainage. The nurse would document the healing process as:

primary intention

Which substance would not be found in glomerular filtrate?

protein

A adult has been diagnosed with a gastroesophageal reflux disease, in which the function of his lower esophageal sphincter is compromised. Which consequence of this condition is most likely?

protrusion of the stomach or regurgitation of stomach contests into the esophagus.

Reviewing pathology for an exam on pulmonary vasculature, the nursing student states that blood enters the right side of the heart via the vena cava, then to the right atrium, right ventricles, and then which vessel carries the deoxygenated blood into the pulmonary system?

pulmonary artery

A client has been diagnosed with deep vein thrombosis (DVT). The nurse is planning care and recognizes that the client is most at risk for:

pulmonary embolism

A client has been diagnosed with deep vein thrombosis (DVT). The. nurse is planning care and recognizes that the client is most at risk for:

pulmonary embolism

The heart is a four-chambered pump. What is the function of the right ventricle?

pumps blood to the lungs

A pregnant client is concerned about her baby having abnormal thyroid function since it "runs in the family." The nurse advises the client to avoid exposure to which teratogenic agent?

radioactive iodine

A nurse preceptor is evaluating the skills of a new registered nurse (RN) caring for a clients experiencing shock. Which action by the new RN indicates a need for more eduction?

raising the head of the bed to a high Fowler's position

A woman in her 28th week of pregnancy tests positive for gestational diabetes mellitus and begins to follow a nutritional plan at home. What result at the follow-up visit indicates a successful outcome?

random blood glucose 85 mg/dl

The nurse is explaining to a client the adverse side effects of cancer therapies. Which target of both chemotherapy and radiation treatment accounts for the adverse as well as therapeutic effects of these treatments?

rapidly proliferating cells

An older adult client has been hospitalized for the treatment of acute pyelonephritis. Which characteristic of the client is most likely implicated in the etiology of this current health problem?

recently had a urinary tract infection

Neurotransmitters exert their actions through specific proteins that re known as:

receptors

A client in the acute stage of inflammation will experience vasodilation of the arterioles and congestion in the capillary beds. The nurse would assess the client's skin for:

redness

A client has had an acute myocardial infarction (MI). The brother of the client has a history of angina. The client asks how he will know if his brother's pain is angina or if the brother is actually having an MI. Which statement is correct?

rest and intake of nitroglycerin relieve chest pain with angina; they do not relieve chest pain with an MI.

After many years of cigarette smoking, a client is admitted to have a "mass" removed from the lung. When explaining the surgery and recovery, the physician notes that the client is likely to have a good amount of fibrosis develop at the surgical area. After the physician leaves the room, the client asks the nurse what was meant by "fibrosis" in the lung. The nurse bases the response on the fact that tissue repaid can:

result in replacement tissue in the form of connective fibrous tissue, which leads to scar formation or fibrosis of the lung.

Which sequence is the correct pathway for blood flow through the heart?

right atrium - tricuspid valve - right ventricle - pulmonary artery - lungs - pulmonary vein - left atrium - mitral valve - left ventricle - aorta

Which health problem may be identified by a TORCH screening test?

rubella and herpes

A 20-year-old college student being treated for a kidney infection developed a temperature of 104 in spite of treatment with antibiotics. Her pulse was high, her blood pressure was low, and her skin was hot, dry, and flushed. The nurse knows that this client most likely is experiencing which type of shock?

septic

The GFR is considered to be the best measure of renal function. What is used to estimate the GFR?

serum creatinine

A child struggling to breathe is brought to the emergency department with a prolonged bronchospasm and severe hypoxemia. Assessment revealed the use of accessory muscles, a weak cough, audible wheezing sound, moist skin, and tachycardia. What is the most likely diagnosis?

severe asthma attack

An 86-year-old female client has been admitted to the hospital for the treatment of dehydration and hyponatremia after she curtailed her fluid intake to minimize urinary incontinence. The client's admitting laboratory results are suggestive of prerenal failure. The nurse should be assessing this client for which early sign of prerenal injury?

sharp decrease in urine output

A child is brought to the emergency department with an asthma attack. Assessment revealed the use of accessory muscles, a weak cough, audible wheezing sound, moist skin, and tachycardia. Which drug will the nurse anticipate administering first?

short-acting beta 2-adrenergic agonists such an albuterol (SABA)

Which type of epithelium is found in the lining of blood vessels, lymph nodes, and alveoli of the lungs?

simple squamous epithelium

Which of the following is called the pacemaker of the heart?

sinoatrial (SA) node

The effective circulating volume is the major regulator of water balance in the body. What else does it regulate?

sodium

The condition of a client with metabolic acidosis fro an intestinal fistula is not improving. The pulse is 125 beats/min and the BP 84/56 mm Hg. ABG values are: pH 7.1, HCO3- 18mEq/L (18 mmol/L), PCO2 57 mm Hg (7.58 kPa). What IV medication should the nurse expect to provide next?

sodium bicarbonate

When red blood cells age, which organ is responsible for their destruction?

spleen

The nursing student, while studying anatomy and physiology, correctly identifies which of the following to be responsible for carrying urine to the bladder?

ureters

The nurse taking a report on a client coming into the emergency room plans care for a client with brain dysfunction based on which symptom?

stupor

The MRA scan of a client with a suspected stroke reports ruptured berry aneurysm. The nurse plans care for a client with:

subarachnoid hemorrhage

The nurse develops a plan to prevent atelectasis in a post surgical client. Which intervention will be effective?

supervision of hourly deep-breathing exercises

A neonate delivered a 36 weeks' gestation is in respiratory distress and requires ventilation. Tests reveal prematurity of the lungs and an underdevelopment of type II alveolar cells. Based on these results, the nurse will anticipate the administration of which of the following to improve respiratory function?

surfactant

The nurse is aware that some drugs may be prevented from entering the brain as a function of:

the blood brain barrier

The nurse is teaching a client with diabetes about the signs and symptoms of hypoglycemia. The client asks, "why will I get headache, disturbed behavior, coma, and seizures if it's my pancreas that's impaired?" which response is the best explanation?

the brain relies on blood glucose as its main energy source.

What happens when a cell is confronted with a decrease in work demands?

the cell becomes smaller.

A client's history and physical indicate that the client is a carrier of sickle cell anemia. The nurse anticipates which of the following?

the client can transmit the disorder but does not exhibit symptoms.

A client experiencing an acute stress response has increased urination. What does the nurse determine is the cause of this clinical manifestation?

the client has hyperglycemia

A client is managing diabetes with exercise and diet. The health care provider reviews the client's most recent lab results: fasting blood glucose level at 80 mg/dL (4.44 mmol/L). and a hemoglobin A1c of 5% (0.05). Select the response that best identifies the client.

the client is achieving normal glycemic control.

The nurse is educating a newly diagnosed client with Hashimoto thyroiditis who is to be discharged from the acute care facility. What should the nurse be sure to include in the education to prevent complications?

the client should be informed about the signs and symptoms of severe hypothyroidism and the need for early intervention.

The postoperative client has a catheter in his brachial artery for continuous blood pressure monitoring. Which assessment would be a concern for the nurse?

the client states his hand is numb

What is the nurse's expectation about a client's ability to compensate for a metabolic blood gas disorder?

the client will compensate with the respiratory system.

A nurse is providing care for an older, previously healthy adult male who has been diagnosed today with pneumococcal pneumonia. Which sign/symptoms is the nurse most likely to encounter?

the client will have a cough producing clear sputum, and he will have faint breath sounds and fine crackles.

An adult client has been admitted to a rehabilitation center after hospital treatment for an ischemic stroke. Which aspect of the client's history would be considered to have contributed to his stroke?

the client's blood pressure has historically been in the range of 150/90 to 160/100 mm Hg.

The health care provider is reviewing lab results of a client diagnosed with heart failure. The provider notes that the client's ANP and BNP levels have been increasing and remain significantly elevated. These results would be interpreted as:

the condition is getting progressively worse.

The nurse is providing care for a client with a diagnosis of cirrhosis, and she notes that the client's sclerae are jaundiced. The nurse recalls that jaundice is a pigment that can accumulate in which part of the cell?

the cytoplasm

The nursing instructor is teaching the students about rheumatic fever. She tells the students that it is an important cause of heart disease and is very serious mainly for which reason?

the disabling effects that result from involvement of heart valves

Potassium is the major cation in the body. It plays many important roles, including the excitability of nerves and muscles. Where is this action particularly important?

the heart

A newborn is screened for congenital hypothyroidism and is found to have the disorder. When educating the mother about the importance of the infant's taking thyroid hormone supplement, what should be included in the education?

the infant will have dosage levels adjusted as he grows.

a 51-year-old male have been diagnosed with alcohol-induced liver disease. He admits to the nurse he does not understand what the liver does in the body. Which of these statements best explains the liver's function?

the liver metabolizes most components of food and also cleans the blood of bacteria and drugs.

A patient is managing his type 2 diabetes with exercise and diet. He has a fasting blood sugar level (FBS) of 80 mg/dL and a hemoglobin A1C of 5%. Based on these findings, which of the following can the nurse assume?

the patient is achieving normal glycemic control.

A 42-year-old male client recently diagnosed with liver cancer is noted as at high risk for bleeding abnormalities. The nurse recognizes this risk as a result of:

the reduction of clotting factors synthesized in the liver.

A patient is admitted with lower urinary tract obstruction and stasis. Which of the following is the primary intervention?

urinary catheterization

A 62-year-old male collapsed while unloading a truck of heavy sacks of feed for his cattle. When he arrived in the emergency department, blood gases revealed a slightly acidic blood sample. The nurse caring for this client is not surprised with this result based on which pathophysiologic rationale?

the skeletal muscles are producing large amounts of lactic acid and release it into the bloodstream during heavy work/exercise.

What determines the number of mitochondria in a given cell type?

the type of activity the cell performs.

A client has been diagnosed with aortic stenosis and asks the nurse what this means. The most appropriate response would be:

the valve opening is narrowed and produces increased resistance to blood flow out of the left ventricle and into the aorta.

A young woman presents with signs and symptoms of urinary tract infection (UTI). The nurse notes that this is the fifth UTI in as many months. What would this information lead the nurse to believe?

there is possible obstruction in the urinary tract

Heart failure and circulatory shock are both conditions of circulatory system failure. Which statement regarding these conditions is correct?

they have the same compensatory mechanisms

The nurse is caring for a 30-year-old woman who is diagnosed with iron deficiency anemia. The client is confused how this condition developed because she ate many foods high in iron. How should the nurse respond?

this iron deficiency anemia was most likely the result of chronic blood loss over a period of time.

The nurse is teaching a new diabetic about fingerstick blood glucose testing. When the client obtains a pre-meal fingerstick reading of 206 mg/dL, which teaching by the nurse is most appropriate?

this is an elevated reading - lets talk about adherence to insulin and diet.

The most common cause of ischemic stroke is:

thrombosis

A client who has just undergone a thyroidectomy is experiencing high fever, tachycardia, and extreme restlessness. The nurse would interpret these findings as manifestations of which complication?

thyroid storm

A client who is scheduled to have a tissue specimen removed for microscopic study will undergo which test?

tissue biopsy

When the mother of an adolescent with Turner syndrome asks the nurse why the physician is prescribing estrogen therapy, the nurse provides which explanation?

to promote secondary sex characteristics

The nurse explains to a client in labor who has demonstrated ineffective contractions impeding progression of labor that the health care provider has added oxytocin infusion to the orders .Which of these does the nurse teach the client is the purpose of oxytocin?

to stimulate contraction of the uterus

The pregnant client asks if there are any precautions she should take in caring for the family cat. The nurse explains that she should limit contact with the cat's litter box. The litter box may contain which organism that can cause fetal malformation?

toxoplasmosis

A client who is being seen in the outpatient clinic reports a single episode of unilateral arm and leg weakness and blurred vision that lasted approximately 45 minutes. The client is most likely experiencing:

transient ischemic attack (TIA)

A client with a suspected MI is brought to the emergency department by ambulance. The nurse caring for this client would expect to receive an order for which laboratory test to confirm a diagnosis of MI?

troponin level

T or F: Malabsorption syndrome results in loss of fat in the stool and failure to absorb fat-soluble vitamins.

true

Which blood vessel layer is made primarily of muscle?

tunica media

A client is told that she has cardiac valve leaflets, or cusps, there are floppy and fail to shut completely, permitting blood flow even when the valve should be completely closed. The nurse knows that this condition can lead to heart failure and is referred to as:

valvular regurgitation

Levels of endothelins may be increased in clients with heart failure. Which of the following is the primary action of endothelins?

vasoconstriction

A client in the hospital following a repair of a left hip fracture is refusing to wear the intermittent pneumatic compression stockings ordered by the physician. The nurse explains to the client that the compression stockings are essential in preventing:

venous thrombosis

The cardiac cycle describes the pumping action of the heart. Which statement is correct about systole?

ventricles contract and blood is ejected from the heart

A 21-year-old male is brought to the ED following a night of partying in his fraternity. His friends found him "asleep" and couldn't get him to respond. They cannot recall how many alcoholic beverages he drank the night before. While educating a student nurse and the man's friends, the nurse begins by explaining the alcohol is:

very lipid-soluble and rapidly crosses the blood-brain barrier.

Select the first stage of hemostasis.

vessel spasm

The nurse is caring for a client who is a strict vegetarian; the client is at greatest risk for the development of:

vitamin B12 deficiency anemia

The nurse is caring for a client with diabetes who has developed gastroparesis. Which of these symptoms does the nurse expect the client to report?

vomiting after eating

The nurse is educating a group of school-age students about the prevention of cold viruses and staying healthy during the school year. What should the nurse inform the students is one of the most important ways for them to avoid spreading a cold?

wash your hands after touching surfaces prior to touching your nose or eyes.

The 40-year-old nurse is concerned that job requires her to stand most of the day. Which therapy could be implemented to prevent tissue injury?

wearing correctly fitted, elastic support stockings

Which procedure reduces the potential for infection primarily by addressing the portal of entry?

wearing gloves when contact with blood or body fluids is anticipated.

A Telehealth nurse is talking with a client who has a history of right-sided heart failure. The nurse should question the client about which assessment finding that would indicated the client's condition is worsening?

weight gain

Which clinical manifestation would lead the nurse to suspect a client has developed fluid volume excess?

weight gain

The nurse is caring for an infant admitted to the emergency department diagnosed with bronchiolitis. The nurse would asses the client for:

wheezy cough and dyspnea

In myasthenia gravis, periods of stress can produce myasthenic crisis. When does myasthenic crisis occur?

when muscle weakness becomes severe enough to compromise ventilation

A high school teenager comes to the emergency room with symptoms that include the abrupt onset of severe pharyngitis and a high fever. The teenager reports that in the previous 4 days he has "just not felt well." The nurse anticipates orders to include which of the following?

white blood cell count

The nurse is reviewing the complete blood count (CBC) and white blood cell (WBC) differential of a client admitted with lower right abdominal pain. Which laboratory results are the most important for the nurse to communicate to the health care provider?

white blood cells (WBCs) 18,500/uL (18.50 x 10 9/L)

A nurse is caring for a client who has sustained severe trauma and has developed disseminated intravascular coagulation (DIC). The nurse will explain this complication to the family based on which of the physiologic principles?

widespread coagulation and bleeding in the vascular compartment

A nurse is caring for a client who has sustained severe trauma and has developed disseminated intravascular coagulation (DIC). The nurse will explain this complication to the family based on which of the physiologic principle?

widespread coagulation and bleeding in the vascular compartment.

Prior to undergoing diagnostic testing with contrast, it is recommended that older adult clients have their creatinine level checked. The rationale for this is to ensure the client:

will not undergo an acute kidney injury by decreasing renal blood flow


Conjuntos de estudio relacionados

national government chapter civil liberties

View Set

Unit: 2. FRACTIONS Assignment: 1. Fractions and Mixed Numbers

View Set

Chapter 4- The Market Forces of Supply and Demand

View Set

Lab: Population Growth and Resource Consumption

View Set

Service and Production Operations Test 2

View Set